Q17

 
jennifer
Thanks Received: 0
Elle Woods
Elle Woods
 
Posts: 91
Joined: July 29th, 2010
 
 
 

PT 49, S3, Q17, P3; Which of the following could most

by jennifer Tue Sep 28, 2010 8:18 am

17. Would it be fair to say that questions such as this one about the final paraphgraph are main point questions?
User avatar
 
bbirdwell
Thanks Received: 864
Atticus Finch
Atticus Finch
 
Posts: 803
Joined: April 16th, 2009
 
This post thanked 1 time.
 
 

Re: PT 49, S3, Q17; Which of the following could most logically

by bbirdwell Thu Sep 30, 2010 11:39 am

In a way, yes, but they're more like the main point of the paragraph than the main point of the passage. Think of it as an inference question. Take the preceding statements into account and make the next logical step -- baby step.
I host free online workshop/Q&A sessions called Zen and the Art of LSAT. You can find upcoming dates here: http://www.manhattanlsat.com/zen-and-the-art.cfm
 
didi0504
Thanks Received: 0
Forum Guests
 
Posts: 23
Joined: October 20th, 2010
 
 
 

Re: PT 49, S3, Q17; Which of the following could most logically

by didi0504 Tue Nov 16, 2010 12:06 am

Could you please explain this question and the approach you would take to find the answer?
I am having a hard time answering this type of questions.

Thanks!
User avatar
 
bbirdwell
Thanks Received: 864
Atticus Finch
Atticus Finch
 
Posts: 803
Joined: April 16th, 2009
 
This post thanked 2 times.
 
 

Re: PT 49, S3, Q17; Which of the following could most logically

by bbirdwell Wed Nov 17, 2010 10:55 am

My approach would be just what I stated above. It's essentially an inference question, so I would put together the important information, taking into account the context of the passage, and make a tiny, baby step forward, almost restating what has already been said.

Here, the paragraph says that male and female prescriptions were quoted indiscriminately, and that no special comment was made about the female writings.

The passage is about women doctors. A logical statement to follow the paragraph would say something like "the fact that women were doctors was not a big deal."

Answer choice (B) says exactly this, except it brings up Plato. So then I go back to double check and make sure Plato thought the same thing. Sure enough, he did (paragraph 2).

(A) is not even close. There is no biographical information about the women.
(C) is the opposite of what we want.
(D) is unsupported.
(E) is also opposite, in a way. The picture is not conflicting. All the evidence cited in the passage is on the same side of the argument.
I host free online workshop/Q&A sessions called Zen and the Art of LSAT. You can find upcoming dates here: http://www.manhattanlsat.com/zen-and-the-art.cfm